Enter your answer and show all the steps that you use to solve this problem in the space provided. Solve the following equation. 6 = x + 2/3

Answers

Answer 1

Answer:

x = 2

Step-by-step explanation:

6 = x + 2/3

-2/3 -2/3

2 = x

Hope this helps! :)

Answer 2
Answer: 16/3

Explanation:
2/3 - 6 = x + 2/3 - 2/3
x = 16/3

Related Questions

A book store is keeping track of the price of books according to the number of pages. Which scatterplot represents that bivariate data? A graph has price (dollars) on the x-axis and pages on the y-axis. A graph has books on the x-axis and pages on the y-axis. A graph has pages on the x-axis and price (dollars) on the y-axis. A graph has books on the x-axis and price on the y-axis.

Answers

Answer:

The graph has pages on the X-axis and price (dollars) on the Y-axis

Step-by-step explanation:

The X-axis will always have the independent variable (pages), while the Y-axis will have the dependent variable (number of pages).  The pages will exist no matter how many there are, but the price is influenced by the number of pages.

Independent variable the graph x dollars y

Find the missing dimensions

Answers

Answer:

short base: 6 yards

long base: 8 yards

Step-by-step explanation:

Our understanding of your figure is shown below.

The question says the "shortest side" and the "width" have the same dimension. If the "width" is a reference to "height 6 yards", then it seems the "shortest side" is 6 yards. Since the slant sides are longer than the height, the "shortest side" is also the "short base."

The short base is 6 yards.

__

The long base overhangs the short base by 1 yard on either end, so it is a total of 2 yards longer than the short base. It it 6+2 = 8 yards long.

The long base is 8 yards.

Hope this is correct, if not then feel free to let me know. I'm sorry in advance if it is incorrect.

6 yards and 8 yards!!!

3.3 + 10m = 7.81 find the value of m ill give brainliest

Answers

Answer:

m = .451

Step-by-step explanation:

3.3 + 10m = 7.81

Subtract 3.3 from each side

3.3-3.3 + 10m = 7.81-3.3

10 m =4.51

Divide each side by 10

10m/10 = 4.51/10

m = .451

Answer:

0.451

Step-by-step explanation:

3.3+10m=7.81

10m = 7.81-3.3

10m = 4.51

m = 4.51/10

m = 0.451

Find each percent of change. State whether the percent increase or decrease. Please show your work.
A) Original : 55 New : 75
B) Original : 233 New : 200

Answers

A: 36.36%
B: 116.5%

Step by step
The answer is is
A: 36.36%
B: 116.5%

Which sentence correctly compares the two points shown on the number line? -1 > 0.25 , -1 < -0.25, -0.25 < -1

Answers

Answer:

-1 < -0.25

Step-by-step explanation:

-1 is less than -0.25

less than symbol = <

greater than symbol = >

Example; -1 > -2 --- negative one is greater than negative two OR -2 < -1 --- negative two is less than negative one

Answer:

-1<-0.25

Step-by-step explanation:

Carla is wrapping a present in the box shown. How much wrapping paper does she need, not including overlap?

Answers

you need to find surface area for this

so the answer is 164in^2

Answer:

164 inches squared

Step-by-step explanation:

Find the surface area.

Add up the area of each side and you get 164 inches squared :)

Together, Matthew and Daniel have 67 DVDs. Matthew’s collection has 3 times as many DVDs as Daniel’s collection. Which equation can be used to determine the number of DVDs in Daniel’s collection?

A. 3x - x = 67
B. x + x = 67
C. 3x + x = 67
D. 3x = 67

Answers

Answer:

C. 3x + x = 67

Step-by-step explanation:

Means Mathew has 3 units and Daniel has 1 unit.

There's 4 units in total.

4x = 67

3x + x = 67

3x is Mathew's DVDs

x is Daniel's DVDs

The answer is c 3x + x =67

(Geometry Question) : Given the points A(-3, -4) and B(2, 0), find the coordinates of the point P on directed line segment AB that partitions AB in the ratio 2:3.

Answers

Answer-7:4


Have a good day
7:4
Mark me Branalist pls

Rule 1: Multiply by 2, then add one third starting from 1. Rule 2: Add one half, then multiply by 4 starting from 0. What is the fourth ordered pair using the two sequences? (5, 10) (four and two thirds, 42) (21, 170) (two and one third, 2)

Answers

2 is the correct answer 2 is the correct answer
the answer your looking for is 2

HELPPPPPPPPPP [WILL GIVE U BRAINLEST AND THANKS]

Answers

The answer to this is 8 1/2

Answer:

C

Step-by-step explanation:

Just + all of the numbers together.

plz correct ans only thx for the help

Answers

Answer:

The width is 5m (A)

Step-by-step explanation:

The formula for area is length times width times height.

LxWxH

I hope this helps!

The area of a floor is 49m2. If the length is 8m, what is the width?
ANSWER: A.) 40 ÷ 8 = 5 m
Explanation:
A floor is a rectangle
The area of a rectangle is length • width
So to find the width, divide the area by the length:
40 ÷ 8= 5 m

A container holds 1 3/4 of litres of water. How many containers can be filled from 49 litres of water?
ANSWER: 28 containers
Explanation: the question is asking how many 1 3/4 liters of water-filled containers will be needed to have a total of 49 litres of water.

49 litres ÷ 1 3/4 litres = 28

ANSWER THIS ASAP WILL MARK THE BRAINLIEST

Answers

Answer:

Option B: 162

Step-by-step explanation:

3^3=27 : top

(3)(3)(6)=54 : middle

(9)(3)(3)=81 : bottom

add all

27+54+81 = 162

I think the answer is c or B

Jamie has a deck of 60 sports cards, of which some are baseball cards and some are football cards. Jamie pulls out a card randomly from the deck, records its type, and replaces it in the deck. Jamie has already recorded nine baseball cards and 11 football cards. Based on these data, what is, most likely, the number of baseball cards in the deck?

A. 20
B. 22
C. 25
D. 27

Please don't just give me an answer please give me a explanation because I'd like to know how to do this equation next time it seems so familiar but I just can't remember besides if you do I'll give you brainly ! = )

Answers

Answer:

If it were like that, 9/20 would be baseball card. 9/20 = X/60

x=27 since 20x3=60

So the answer is 27  

Step-by-step explanation:

27 ajjidng d itenhundubyneiy dutnjd y

SOLVE AND NO LINKS
x – 3 = 5

Answers

Answer:

8 = x

or

x = 8

Step-by-step explanation:

x – 3 = 5

x + -3 + 3 = 5 + 3

x = 5 + 3

(5 + 3 = 8)

x = 8

Answer:

x=8

Step-by-step explanation:

x-3=5

x-3+3=5+3

x=8

What is the volume of the composite figure shown? Use 3.14 for π.

Answers

Answer:

25.12

Step-by-step explanation:

the formula of a cone is pi r^2 * h/3

r = radius and h = height

the radius is 2 and the height is 3

and the question says use 3.14 instead of pi so

we have

3.14 * 2^2 * 6/3

2*2 or 2^2 = 4

3.14 * 4 * 6/3

6/3 = 2

3.14 * 4 * 2

4*2 = 8

and 8 * 3.14 is 25.12

The answer to this question is 25.12

Write an addition or subtraction equation

Answers

Answer:

0-2=-2

Step-by-step explanation:

We start at 0, and go down 2, so we have the equation 0-2=-2

Hope this helps!

--Applepi101

Apples right , you can do that both ways but when you go up it’s positive , but when going below 0 it gets negative

the width w of a rectangular swimming pool is x+5 the area a of the pool is x^2+4-5

what is an expression for the length of the pool? step by step

Answers

Answer:

x-1

Step-by-step explanation:

Area= x^2+4x-5

Width= x+5

Area= Length*breadth

x^2+4x-5=length*x+5

Length=x^2+4x-5/x+5

Therefore, length= x-1

Answer:

x-1

Step-by-step explanation:

i got this problem on khan academy

If a Car can drive 35 miles on 7 gallons of gas, how many miles can it travel on 2 gallons of gas (assuming miles per gallon is proportional)? Give and explain your answer.

Answers

Answer:

10 miles

Step-by-step explanation:

35/7 is 5 so therefore if you do 5+5+5+5+5+5+5 is 35

so that means 2 gallons which is 10

10 miles! if you divide 35 by 7 you get 5 so that mean it drives 5 miles per gallon with 2 gallons it will drive 10 miles

Question on file. Thx in advance

Answers

The answer is 1,320
Expiation: 22x60= 1,320
Since there are 60 min in a hour you must multiply the 22L produced in a minute by 60
The answer is 1,320

How many times as great as
2 x 103 is 8 x 105?

Answers

2 x 103 = 206, 8 x 105 is 840.
840 is roughly 4.08 times greater than 206.

Answer:gấp 103/420

Step-by-step explanation:2*103 =206

8*105=840 rồi lấy 206/840 =103/420 đáp án

how do I graph k(x)=13 x


show work plz.

Answers

Answer:

Step-by-step explanation:

Use the slope and y intercept, or 2 points

Slope: 13

y-int: 0,0

x|y

0|0

1|13

ANSWER- 0|0

Explainion - :))

The prices for 7 different totes are $52,$22, $50, $44, $60, $48, and $52. What is the outlier in the data set?

Answers

Answer:

$22

Step-by-step explanation:

An outlier in a data set is the one farthest from the other data points. 22 is far away from the other points, so that is the outlier.

Answer:

22

Step-by-step explanation:

It is farther off than from all of the rest of the #'s.

surds.... plzz help, matching up, thank you

Answers

1 one to D
2 one to B
3 one to C
4 one to A

Answer:

1 - D

2 - B

3 - C

4 - A

I hope this helped!

What is 50% of 174.95?Thanks.

Answers

87.474 hope it’s right :)
The answer is 87.475

Malik and Janae are practicing making free throws at the basketball court.

Malik makes 3 baskets for every 4 shots he takes.

Janae makes 4 baskets for every 5 shots she takes.

Which number accurately completes the table?

Answers

Answer:

12

Step-by-step explanation:

Hope this is correct, if it isn't then feel free to let me know so i can fix my mistake. I'm sorry in advance if it isn't correct.

12

Step-by-step explanation:

I WAS THE FIRST ONE TO SAY IT!

What is 60,000 + 3,000 + 20 + 9 in standard form

Answers

Answer:

the standard form is 63029

63029 is the answer in standard form

2) Cereal cost $7 / box. How many boxes
did Huong buy if she spent $21?

Answers

Answer:3 boxes.
Explanation: $21 / $7=3 boxes
3 boxes 7X3=21 so there is your answer

The table shows the number of practice problems completed in 30 minutes in three samples of 10 randomly selected math students.
Which statement is most accurate based on the data?

Answers

Answer: A

Step-by-step explanation: As it is a data set that has been equally represented, it is ok to predict.

Answer A step by step explanation as it is set that has been equaling resendted it is ok to predict

show all your work please and thank you let me show you an example of showing your work

the second one is an example from my other one I just asked

Answers

Answer:

so we jsut show the work for the first question I'll write in the comment box then

Answer:

700 cubic inches

Step-by-step explanation:

The question asks for how much volume the box can hold. We multiply the length (14), width (10), and height (5) to get the volume. 14 · 10 · 5 = 700.

The box can hold 700 cubic inches.

the mark obtained by 20 students in mathematics in the examination are given below.

group the data into the class interval of length 10 and construct a frequency distribution.

table with tally marks

27, 38,25,18,9,24, 48, 15,27,35

23,45,32,16,26,39,20,33,40,37

Answers

Answer:

Step-by-step explanation:

class interval

9-1920-2930-3940-49

Frequency

5762

tally

IIIIIIIIII   IIIIIII   III

Note: the fifth tally marks crosses the first 4 marks

I honestly just need the points thankyou
Other Questions
Activated macrophages and neutrophils release cytokinin, signaling molecules that enhance the immune response Select one: True False find the area of irregular figures I wanna make a podcast about future. Can you give me some questions about future and some answer? Please I need this for my school project On August 4, Armstrong Trucking, Inc., paid $4,500 to replace the engine in one of its trucks. Required:Write the necessary journal entry. Click to read the passage from "The Black Cat," by Edgar Allan Poe. Thenanswer the question.How would a reader most likely feel after reading this paragraph at thebeginning of a story?A. Confused, because the narrator sounds sane, despite claiming notto be.B. Concerned for the narrator's safety, since he claims that someonehas committed a crime against him.C. Intrigued by why the narrator is about to die.D. Curious about why the narrator has admitted to lying. please helppppppppppppppp what is a bank statement The wheel was first documented as being used in Mesopotamia(Iraq today) in 3500 B.C. The first steam powered vehicle thatused wheels was invented in 1672 A.D. How long did it take forthe wheel (from being invented) to get pushed by a machine? Here is some information about a holiday.7 night holiday$340 per person8% discount if you book before 31 MarchOn 15 February, Naseem books this holiday for 2 people.Calculate the total cost of his holiday. Quadrilateral qrst is a square what is pst Help asappppp hbshhshshhshshsshshshs NEED HELP ASAP 50 POINTS WRITING PROMPT: In this unit, you studied the challenges and benefits of 90living in the twenty-first century. What are some of the challenges andbenefits of living in a global society? Do you think there are moreadvantages or disadvantages to modern life as compared to life acentury ago? Why? (Answer must be 3 paragraphs long) Which number is a rational number? What is scrum of scrum The Industrial Revolution caused a change in how the merchant middle class viewedand treated workers. In the table below, compare how merchants and factory owners did business PLEASE I NEED HELP FAST ADN l g ?hihihihihihihihihi Most enzymes can function within a rather broad pH range, approximately 3-12. True or false? Find f(-3) PLEASE HELP When the park opened for the day, four people were in attendance. Every hour, the number of people tripled from the previous hour's attendance.Part A: Determine the common ratio between the attendance each hour.Part B: Write an equation that can be used to determine the attendance for the nth hour the park is open.Part C: How many people were at the park during the second, fifth, and eighth hours the park was open? Show all work.